LSAT and Law School Admissions Forum

Get expert LSAT preparation and law school admissions advice from PowerScore Test Preparation.

User avatar
 Dave Killoran
PowerScore Staff
  • PowerScore Staff
  • Posts: 5853
  • Joined: Mar 25, 2011
|
#88066
Complete Question Explanation
(The complete setup for this game can be found here: lsat/viewtopic.php?f=165&t=7597)

The correct answer choice is (C).

The question stem asks you to identify the greatest number of messages between J’s first message and H’s first message. As shown in Templates 2B (J-L-L-H-J) and 3B (J-L-L-H-H-J), there can be two messages between J’s first message and H’s first message. Thus, answer choice (C) is correct.

Get the most out of your LSAT Prep Plus subscription.

Analyze and track your performance with our Testing and Analytics Package.